mock Exam 2017, answers PDF

Title mock Exam 2017, answers
Course Number Theory
Institution University of Exeter
Pages 11
File Size 193.7 KB
File Type PDF
Total Downloads 67
Total Views 159

Summary

Download mock Exam 2017, answers PDF


Description

SOLUTIONS ECM3704 UNIVERSITY OF EXETER MATHEMATICS May 2017 Number Theory - SOLUTION SHEET Module Leader: Dr Julio Andrade Duration: 2 HOURS. The mark for this module is calculated from 80% of the percentage mark for this paper plus 20% of the percentage mark for associated coursework.

Answer Section A (50%) and any TWO of the three questions in Section B (25% for each). Marks shown in questions are merely a guideline. Candidates are permitted to use approved portable electronic calculators in this examination. This is a CLOSED NOTE examination.

SECTION A 1. (a) Find the general solution of the following system of simultaneous congruences:   x ≡ 1 (mod 3) x ≡ 2 (mod 5)   x ≡ 3 (mod 7).

SOLUTION: Since the moduli m1 = 3, m2 = 5, and m3 = 7 are pairwise relatively prime, by the Chinese Remainder Theorem (CRT), the linear system has a unique solution. To find it, first we find M1 , M2 , M3 , y1 , y2 , and y3 as in the proof of the CRT. Thus, by the CRT,

x≡

3 X

ai Mi yi

(mod M)

i=1

≡ 1 · 35 · 2 + 2 · 21 · 1 + 3 · 15 · 1 (mod 105) ≡ 52 (mod 105). Therefore, 52 is the unique solution of the linear system modulo 105. Thus, the general solution is x = 52 + 105t. [Standard problem - similar seen in lectures and example sheets] (8) (b) Find all solutions of the congruence f (x) = x2 + 4x + 2 ≡ 0 (mod 49). SOLUTION: Since 49 = 72 , we first solve the congruence f (x) ≡ 0 (mod 7), since any solution to f (x) ≡ 0 (mod 49) must also be a solution to f (x) ≡ 0 (mod 7). By inspection, we find x ≡ 1 (mod 7) and x ≡ 2 (mod 7) are solutions to the congruence modulo 7. We compute ′ f (x) = 2x + 4. Page 2 of 11

ECM3704/continued . . .





In particular, f (1) = 6 6≡ 0 (mod 7) and f (2) = 8 ≡ 1 6≡ 0 ′ (mod 7). We now compute the inverse of f (1) (mod 7) which is f ′ (1) = 6 = 6, ′

and an inverse of f (2) (mod 7) which is f ′ (2) = 1. ′

Since f (1) 6≡ 0 (mod 7) then Hensel’s lemma applies. It tell us that there is a unique solution (mod 49) to f (x) ≡ 0 (mod 49) that is congruent to 1 (mod 7). In fact, the solution is x ≡ 1 − f ′ (1)f (1) ≡ 1 − 6 · 7 ≡ −41 ≡ 8 (mod 49). ′

By the same reasoning as before, since f (2) 6≡ 0 (mod 7), Hensel’s lemma tell us there is a unique solution (mod 49) to f (x) ≡ 0 (mod 49) that is congruent to 2 (mod 7), and this solution is x ≡ 2 − f ′ (2)f (2) ≡ 2 − 1 · 14 ≡ −12 ≡ 37 (mod 49). Therefore we conclude that the solutions are x ≡ 8 (mod 49)

and

x ≡ 37 (mod 49).

[Standard problem - similar seen in lectures and example sheets] (8) (c) State (without proof ) the Euler-Fermat theorem and then find the remainder when 252550 is divided by 18. SOLUTION: The Euler-Fermat Theorem: Let n ∈ N, a ∈ Z and suppose that gcd(a, n) = 1. Then aϕ(n) ≡ 1 (mod n). Since 25 ≡ 7 (mod 18) and ϕ(18) = 6, by the Euler-Fermat theorem, 76 ≡ 1 (mod 18). Therefore, 252550 ≡ 72550 ≡ (76 )425 ≡ 1425 ≡ 1 (mod 18). Thus, the desired remainder is 1. [Standard problem - similar seen in lectures and example sheets] (3 marks for the definition and 5 marks for the remainder) (8) Page 3 of 11

ECM3704/continued . . .

(d) Compute the Legendre symbol 

 31 . 641

SOLUTION: 

31 641



= = = = = = = =



 641 as 641 ≡ 1 (mod 4) 31   21 31    7 3 31 31       31 31 − − as 3, 7, 31 are all ≡ 3 (mod 4) 3 7       3 1 − − 3 7    7 as 3, 7, are both ≡ 3 (mod 4) [−1] + 3   1 − 3 −1.

[Standard problem - similar seen in lectures and example sheets] (8) (e) If n = a2 +b2 , we will regard any of the 8 expressions (±a)2 +(±b)2 and (±b)2 + (±a)2 for n as equivalent to a2 + b2 . Find four inequivalent representations of 112201 = 29 · 53 · 73. SOLUTION: It is easy to see that 112201 = 29 · 53 · 73 2

2

and that 29 = 2 + 5 , 53 = 72 + 22 , and 73 = 82 + 32 . Thus computing real and imaginary parts of (2 + 5i)(2 ± 7i)(3 ± 8i) we obtain that 112201 = 852 + 3242 = 1762 + 2852 = 3002 + 1492 = 992 + 3202 . Page 4 of 11

ECM3704/continued . . .

[Standard problem - similar seen in lectures and example sheets] (8) (f) Let p be a prime such that p ≡ 1 (mod 4). Prove that [((p − 1)/2)!]2 ≡ −1 (mod p). SOLUTION: Proof: Because p − i ≡ −i (mod p), (p − 1)! = [1 · 2 · · · (p − 1)/2][((p+1)/2) · · · (p−1)] ≡ [(p−1)/2]! {[−(p − 1)/2] · · · (−2)(−1) } ≡ [(p−1)/2]![(p−1)/2]!(−1)(p−1)/2 ≡ (−1)(p−1)/2 {[(p − 1)/2]!}2 (mod p). Therefore, by Wilson’s theorem, (−1)(p−1)/2 {[(p − 1)/2]!}2 ≡ −1 (mod p); that is, {[(p − 1)/2]!}2 ≡ (−1)(p+1)/2 (mod p). Since p ≡ 1 (mod 4), (p + 1)/2 is odd, so {[(p − 1)/2]!}2 ≡ −1 (mod p). (10) [Unseen, but easy application of theorems from lectures]

Page 5 of 11

ECM3704/continued . . .

[50]

SECTION B 2. Throughout this question, you may use about  standard   facts   the −1 2 3 Legendre symbol, including the values of , and . p p p  m valid for m ∈ Z (a) Give a careful definition of the Jacobi symbol n   2 and odd positive integers n. Prove that the Jacobi symbol n equals 1 if and only if n ≡ ±1 (mod 8). (You may assume that the definition of the Legendre symbol is known). SOLUTION: Definition (Jacobi Symbol): Let n = p1 . . . pr be the factorization of n into not necessarily distinct odd primes. Then m n

:=

 r  Y m i=1

pi

,

where the symbols on the right are Legendre symbols. We also define a1 = 1. Write n = p1 p2 . . . pr where the odd prime factors pi are not necessarily distinct. Then we have

n2 =

r Y

(1 + pi2 − 1) = 1 +

r X

(p2i − 1) +

i6=j

i=1

i=1

X 2 (pi − 1)(pj2 − 1) + · · · .

Since each pi is odd, we have p2i − 1 ≡ 0 (mod 8) so r X (pi2 − 1) n ≡1+ 2

(mod 64)

i=1

hence

r

X1 1 2 (n − 1) ≡ (pi2 − 1) 8 8 i=1

(mod 8).

This also holds modulo 2, hence

Page 6 of 11

ECM3704/continued . . .

 Y   Y r r  2 2 2 2 (−1)(pi −1)/8 = (−1)(n −1)/8 . = = n pi i=1 i=1

As n is odd we must have n ≡ ±1, ±3 (mod 8) and by checking the cases n = ±1, ±3 we deduce that n2 − 1 ≡ 8

( 0 (mod 2), n ≡ ±1 (mod 8) 1 (mod 2), n ≡ ±3 (mod 8),

which completes the proof. [Definition and Proof given in lectures] (3 marks for the definition and 7 marks for the proof) (10) (b) State (without proof ) the Reciprocity Law for Jacobi symbols. SOLUTION: The Reciprocity Law for Jacobi Symbols. Let m and n be coprime odd positive integers. Then m  n  n

m

= (−1)(m−1)(n−1)/4

( +1, if m ≡ 1 (mod 4) or n ≡ 1 (mod 4), = −1, if m ≡ n ≡ 3 (mod 4).

[Theorem stated (and proven) in lectures]

(5)

(c) Let m be an  3odd  positive integer such that 3 does not divide m. Prove that m = 1 if and only if the sum of the exponents of the prime factors congruent to ±5 (mod 12) of m is even. SOLUTION: Qk ei Qk (3/pi )ei . Since pi . Then (3/m) = i=1 Let m = i=1 ( 1, if p ≡ ±1 (mod 12) (3/p) = −1 if p ≡ ±5 (mod 12), we need only consider the case when m contains prime factors ≡ ±5 (mod 12). Each such factor p contributes a −1 to the product, e so (3/p)P contributes (−1)e to the product. Thus (3/m) = 1 if and only if e is even. [Unseen, but easy consequence of standard results] (5)

(d) Let p be an odd prime. Prove that p−1   X j =0 p j=1 Page 7 of 11

ECM3704/continued . . .

where

  j p

is the Legendre symbol.

SOLUTION: By the Proposition 3.4 from the lecture notes, precisely half of the integers in the interval [1, p − 1] are quadratic residues (while the Pp−1  j  other half are non-residues). Therefore in the sum j=1 p half of the terms are equal to 1 and half are equal to −1, so the sum is equal to 0. [Unseen, but easy application of results seen in class] (5) [25] 3. (a) Prove that 30 | (n5 − n), where n is an arbitrary integer. SOLUTION: We have that n5 − n = n(n4 − 1) = n(n2 − 1)(n2 + 1) = n(n − 1)(n + 1)(n2 − 5n + 6 + 5n − 5) = n(n − 1)(n + 1)[(n − 2)(n − 3) + 5(n − 1)] = (n − 3)(n − 2)(n − 1)n(n + 1) + 5(n − 1)2 n(n + 1). The first term on the RHS is the product of five consecutive integers, so it is divisible by 5! = 120. Since 3! | (n − 1)n(n + 1), the second term on the RHS is divisible by 5 · 6 = 30. Thus the RHS is divisible by min(120, 30) = 30. Consequently, 30 | (n5 − n) for every n. [Unseen, application of concepts seen in class] (7) (b) Prove that there is no polynomial f with integer coefficients such that the value f (n) is a prime for every integer n. SOLUTION: Suppose there is such a polynomial f (n) = ak nk + ak−1 nk−1 + · · · + a1 n + a0 , where ak 6= 0. Let b be some integer. Since f (n) is always a prime, f (b) must be a prime p; that is, f (b) = ak bk + · · · + a1 b + a0 = p.

(1)

Let t be an arbitrary integer. Then

Page 8 of 11

ECM3704/continued . . .

f (b + tp) = ak (b + tp)k + ak−1 (b + tp)k−1 + · · · + a1 (b + tp) + a0 = (ak bk + ak−1 bk−1 + · · · + a1 b + a0 ) + p · g(t) where g(t) is a polynomial in t. Thus, by (1) f (b + tp) = p + pg (t) = p[1 + g(t)]. So p | f (b + tp). But every value of f is a prime, so f (b + tp) must be a prime and hence f (b + tp) = p. Thus, f (b) = p = f (b + tp). This implies f takes on the same value infinitely many times, since t is an arbitrary integer. But f (n) is a polynomial of degree k, so it cannot assume the same value more than k times, yielding a contradiction. Thus, no polynomial with integral coefficients exists that will generate only primes. [Unseen, application of concepts seen in class] (10) (c) Prove that an odd integer n > 1 is a prime if and only if it is not expressible as a sum of three or more consecutive positive integers. SOLUTION: If an odd integer n is expressible as a sum of three or more consecutive positive integers, then for some m ≥ 1 and k ≥ 3, n = m + (m + 1) + . . . + (m + (k − 1)) = km +

k(k − 1) . 2

If k is odd, then n = k(m + k−1 ) and cannot be prime (k and 2 k−1 m + 2 are integers strictly bigger than 1). If k is even, then, n = k2 (2m + (k − 1)) and once again cannot be prime as k/2 and 2m + (k − 1) are integers strictly bigger than 1. Conversely, if an odd integer n is not prime, write n = ab for some other positive integers a and b strictly bigger than 1. Then a and b must be odd. Assume a ≤ b without loss of generality. Let k = a ≥ 3 and m = b − a−1 ≥ a − a−1 = a+1 ≥ 2. Then, 2 2 2 m+(m+1)+· · ·+(m+(k−1)) = km+

k−1 k(k − 1) ) = ab = n. = k(m+ 2 2

[Problem similar to the ones in Example sheets]

(8) [25]

Page 9 of 11

ECM3704/continued . . .

4. (a) Prove that if n ≡ 3 (mod 4), then n is not representable as the sum of two squares. SOLUTION: Suppose n = x2 + y 2 for some integers x and y. Since w2 ≡ 0 or 1 (mod 4) for any integer w, it follows that n = x2 + y 2 ≡ 0, 1 or 2 (mod 4). Since n ≡ 3 (mod 4), the result follows. [Unseen, but easy application of concepts and results seen in class] (5) (b) Let p be a prime. Prove that if p ≡ 1 (mod 4), then there are positive integers x and y such that x2 + y 2 = kp for some positive integer k, where k < p. SOLUTION: Since p ≡ 1 (mod 4) and that −1 is a quadratic residue of p we have that there is a positive integer a < p such that a2 ≡ −1 (mod p); that is, a2 + 1 = kp for some positive integer k. Thus, there are integers x = a and y = 1 such that x2 + y 2 = kp. It remains to show that k < p. Since a ≤ p − 1 and kp = a2 + 1 < (p − 1)2 + 1, kp < p2 − 2(p − 1); so kp < p2 and hence k < p, as desired. [Similar to results seen in class] (8) (c) Prove that every prime p ≡ 1 (mod 4) can be written as the sum of two squares. SOLUTION: By part (b) and the well-ordering principle we obtain that there is a least positive integer m such that mp = x2 + y 2 for some suitable positive integers x and y. We shall show that m = 1 by contradiction. Suppose m > 1. Define integers r and s by

r ≡ x (mod m)

and

s≡y

(mod m)

(2)

where −m/2 < r, s ≤ m/2.

(3)

Then r 2 + s2 ≡ x2 + y 2 = mp ≡ 0 (mod m), so r 2 + s2 = mn for some positive integer n. Therefore, (r 2 + s2 )(x2 + y 2 ) = (mn)(mp) = m2 np. But, using the result (seen in lectures) Page 10 of 11

ECM3704/continued . . .

that the product of two sums of two squares is representable as the sum of two squares, we have that (r 2 + s2 )(x2 + y 2 ) = (rx + sy)2 + (ry − sx)2 . Thus, (rx + sy)2 + (ry − sx)2 = m2 np.

(4)

It follows by congruences (2) that rx+sy ≡ x2 +y 2 ≡ 0 (mod m)

and

ry −sx ≡ xy−yx ≡ 0 (mod m).

These two congruences imply that both (rx + sy)/m and (ry − sx)/m are integers. It now follows from (4) that np =



rx + sy m

2

+



ry − sx m

2

which is a sum of two squares. The inequalities (3) imply that mn = r 2 +s2 ≤ (m/2)2 + (m/2)2 = m2 /2. Thus, mn ≤ m2 /2; that is, n ≤ m/2. Therefore, n < m. Now n 6= 0, since if n = 0, then r 2 + s2 = 0; so r = 0 = s. Then x ≡ 0 ≡ y (mod m), so m | x and m | y. Therefore, m2 | (x2 +y 2 ): that is, m2 | mp. This implies m | p. Since m < p by part (ii), m must be 1. This negates our hypothesis that m > 1. Therefore n ≥ 1. Thus, n is a positive integer < m such that np is a sum of two squares. This is a contradiction, since we assumed that m is the least such positive integer. Consequently, m = 1; that is, p = x2 + y 2 , as desired. [Proven in class] (12) [25]

Page 11 of 11

ECM3704/END OF PAPER...


Similar Free PDFs